Középiskolai Matematikai és Fizikai Lapok
Informatika rovattal
Kiadja a MATFUND Alapítvány
Már regisztráltál?
Új vendég vagy?

A 2003 októberi A-jelű matematika feladatok megoldása

A közöltek csak megoldásvázlatok, esetleg csak végeredmények. A maximális pontszám eléréséhez általában ennél részletesebb megoldás szükséges. A részletes megoldásokat a beküldött dolgozatok alapján a KöMaL-ban folyamatosan közöljük.


A. 326. Legyenek x1,x2,...,xn olyan egész számok, amelyeknek nincs 1-nél nagyobb közös osztója, továbbá tetszőleges k pozitív egészre legyen

sk=x1k+...+xnk.

Bizonyítsuk be, hogy az 1,2,...,n számok legkisebb közös többszöröse osztható az s1,s2,...,sn számok legnagyobb közös osztójával.

Megoldás. Legyen \(\displaystyle p^\alpha\) az s1,...,sn számok tetszőleges közös prímhatvány osztója. Azt kell megmutatnunk, hogy \(\displaystyle p^\alpha\le n\).

Először is megmutatjuk, hogy tetszőleges k pozitív egész esetén \(\displaystyle p^\alpha|s_k\) (tehát k>n esetén is). Legyen f(t)=tn+a1tn-1+...+an-1t+an az a polinom, amelynek gyökei az x1,...,xn számok. Ez egy egész együtthatós polinom, és tetszőleges k>n esetén

\(\displaystyle s_k+a_1s_{k-1}+\dots+a_ns_{k-n}=\sum_{i=1}^nx_i^{k-n}f(x_i)=0.\)

Ezért, ha sk-1,...,sk-n mindegyike osztható \(\displaystyle p^\alpha\)-val, akkor sk is osztható vele. Mivel az s1,...,sn számok ilyenek, ebből indukcióval következik az állítás az összes pozitív egész k-ra.

Legyen most k egy olyan szám, amely nagyobb \(\displaystyle \alpha\)-nál és osztható \(\displaystyle \varphi(p^\alpha)\)-val. Ekkor tetszőleges x egész számra \(\displaystyle x^k\equiv0\pmod{p^\alpha}\) vagy \(\displaystyle x^k\equiv1\pmod{p^\alpha}\) attól függően, hogy x osztható-e a p prímszámmal. Mivel \(\displaystyle p^\alpha|s_k=x_1^k+\dots+x_n^k\), az x1,...,xn számok között a p-vel nem oszthatók száma osztható \(\displaystyle p^\alpha\)-val. Mivel lnko(x1,...,xn)=1, legalább egy ilyen xi van. Mindebből következik, hogy a p-vel nem osztható xi száma legalább \(\displaystyle p^\alpha\), ezáltal \(\displaystyle p^\alpha\le n\).


A. 327. Az n-edfokú, valós együtthatós

p(x)=anxn+an-1xn-1+...+a1+a0

polinom (n\(\displaystyle \ge\)3) mindegyik (valós és komplex) gyöke a bal félsíkban van, azaz negatív a valós része. Igazoljuk, hogy tetszőleges 0\(\displaystyle \le\)k\(\displaystyle \le\)n-3 esetén

akak+3<ak+1ak+2.

IMC 10, Kolozsvár, 2003

Megoldás. Az f polinom első- és másodfokú tényezőkre bontható: \(\displaystyle f(z)=\prod_i(k_iz+l_i)\cdot\prod_j(p_jz^2+q_jz+r_j)\), ahol ki, li, pj, qj, rj valós számok. Mivel minden gyök a bal félsíkban van, minden egyes i-re ki és li, illetve pj, qj és rj előjele megegyezik. Tehát, szükség esetén a polinomot (-1)-gyel megszorozva, a gyökök megváltozása nélkül elérhetjük, hogy az együtthatók mind pozitívak legyenek.

Az egyszerűbb leírás kedvéért az együtthatók sorozatát mindkét irányban kiterjesztjük az an+1=an+2=...=0 és a-1=a-2=...=0 értékekkel, és az állítást -1\(\displaystyle \le\)k\(\displaystyle \le\)n-2 esetén bizonyítjuk indukcióval.

Az n\(\displaystyle \le\)2 esetben az állítás triviális: ak+1 és ak+2 pozitív, miközben ak-1 és ak+3 közül legalább az egyik 0; tehát, ak+1ak+2>akak+3=0.

Most tételezzük fel, hogy n\(\displaystyle \ge\)3 és az állítás minden kisebb értékre igaz. Vegyük az f polinom egy z2+pz+q alakú osztóját, ahol p és q pozitív valós számok. (Ilyen osztót egy konjugált gyökpárból kaphatunk, vagy pedig két negatív valós gyökből.) Ekkor

\(\displaystyle f(z)=(z^2+pz+q)(b_{n-2}z^{n-2}+\dots+b_1z+b_0)=(z^2+pz+q)g(x).\eqno(1)\)

A g(z) polinom gyökei is a bal félsíkban vannak, tehát bk+1bk+2<bkbk+3 minden -1\(\displaystyle \le\)k\(\displaystyle \le\)n-4 esetén. Definiálva bn-1=bn=...=0 és b-1=b-2=...=0 értékeket is, minden egész k-ra igaz, hogy bk+1bk+2\(\displaystyle \le\)bkbk+3.

Most igazoljuk, hogy ak+1ak+2>akak+3. Ha k=-1 vagy k=n-2, akkor ez triviális, mert ak+1ak+2 pozitív és akak+3=0. Tehát, tegyük fel, hogy 0\(\displaystyle \le\)k\(\displaystyle \le\)n-3. Mint könnyen ellenőrizhető,

ak+1ak+2-akak+3=

=(qbk+1+pbk+bk-1)(qbk+2+pbk+1+bk)-(qbk+pbk-1+bk-2) (qbk+3+pbk+2+bk+1)=

=(bk-1bk-bk-2bk+1)+p(bk2-bk-2bk+2)+q(bk-1bk+2-bk-2bk+3)+

+p2(bkbk+1-bk-1bk+2)+q2(bk+1bk+2-bkbk+3)+pq(bk+12-bk-1bk+3).

Megmutatjuk, hogy mind a hat tag nemnegatív és legalább az egyikük pozitív. A p2(bkbk+1-bk-1bk+2) tag pozitív, mert 0\(\displaystyle \le\)k\(\displaystyle \le\)n-3. A bk-1bk-bk-2bk+1 és q2(bk+1bk+2-bkbk+3) tagok szintén nemnegatívak.

A p(bk2-bk-2bk+2) szám előjelének vizsgálatához tekintsük a

bk-1(bk2-bk-2bk+2)=bk-2(bkbk+1-bk-1bk+2)+bk(bk-1bk-bk-2bk+1)\(\displaystyle \ge\)0

egyenlőtlenséget. Ha bk-1>0, akkor oszthatunk vele: bk2-bk-2bk+2\(\displaystyle \ge\)0. Ellenkező esetben, ha bk-1=0, akkor bk-2=0 vagy bk+2=0, tehát bk2-bk-2bk+2=bk2\(\displaystyle \ge\)0. Tehát, p(bk2-bk-2bk+2)\(\displaystyle \ge\)0 minden k-ra. Hasonlóan kapjuk, hogy pq(bk+12-bk-1bk+3)\(\displaystyle \ge\)0.

A q(bk-1bk+2-bk-2bk+3) kifejezés előjele hasonlóan vizsgálható.

bk+1(bk-1bk+2-bk-2bk+3)=bk-1(bk+1bk+2-bkbk+3)+bk+3(bk-1bk-bk-2bk+1)\(\displaystyle \ge\)0.

Ha bk+1>0, akkor oszthatunk vele. Ellenkező esetben bk-2=0 vagy bk+3=0. Mindkét esetben bk-1bk+2-bk-2bk+3\(\displaystyle \ge\)0.

Ezzel mind a hat tag előjelét megvizsgáltuk, a bizonyítás kész.


A. 328. Határozzuk meg mindazokat az f: (0,\(\displaystyle \infty\))\(\displaystyle \to\)(0,\(\displaystyle \infty\)) függvényeket, amelyekre tetszőleges x,y>0 esetén

f(f(x)+y)=xf(1+xy).

Megoldás. Megmutatjuk, hogy a függvényegyenletnek egyetlen megoldása van, az f(x)=1/x függvény.

1. állítás. Az f függvény monoton csökken.

Biz. Tegyük fel, hogy valamely 0<u<v esetén f(u)<f(v). Legyen \(\displaystyle w={vf(v)-uf(u)\over v-u}\). Erre a számra igaz, hogy w>f(v)>f(u), mert \(\displaystyle w-f(v)={u(f(v)-f(u))\over v-u}\). Írjuk fel a függvényegyenletet az x=u, y=w-f(u), illetve x=v, y=w-f(v) számpárokra:

\(\displaystyle f(w)=f(f(u)+(w-f(u)))=uf(1+u(w-f(u))) =uf\left(1+{uv(f(v)-f(u))\over v-u}\right);\)

\(\displaystyle f(w)=f(f(v)+(w-f(v)))=vf(1+v(w-f(v))) =vf\left(1+{uv(f(v)-f(u))\over v-u}\right).\)

Ez azonban ellentmondás, mert u\(\displaystyle \ne\)v. tehát u<v esetén f(u)\(\displaystyle \ge\)f(v), a függvény valóban monoton csökken.

2. állítás. f(1)=1.

Biz. Tegyük fel, hogy f(1)\(\displaystyle \ne\)1. A függvényegyenletbe (x=1)-et írva, f(f(1)+y)=f(1+y), azaz f(u+|f(1)-1|)=f(u), ha u>1. Ebből következik, hogy a függvény az (1,\(\displaystyle \infty\)) intervallumban |f(1)-1| szerint periodikus. Ez a monotonitással együtt azt jelenti, hogy konstans. Ekkor viszont x és y helyére 1-nél nagyobb számokat írva a baloldalon f(f(x)+y) konstans, a jobboldalon xf(1+xy) nem konstans, ami szintén ellentmondás. Tehát f(1) nem lehet más, csak 1.

3. állítás. \(\displaystyle f(x)={1\over x}\).

Biz. Legyen először x>1 és \(\displaystyle y=1-{1\over x}\). Ezzel a helyettesítéssel 1+xy=x, és a függvényegyenlet a következőt adja:

\(\displaystyle f\left(f(x)-{1\over x}+1\right)=xf(x).\)

Ha \(\displaystyle f(x\){1\over x}">, akkor \(\displaystyle f(x)-{1\over x}+\)1">, és a függvény monotonitása miatt \(\displaystyle f\left(f(x)-{1\over x}+1\right)\le1\), ugyanakkor a jobboldalon xf(x)>1, ami ellentmondás. Hasonlóan, ha \(\displaystyle f(x)<{1\over x}\), akkor \(\displaystyle f(x)-{1\over x}+1<1\), a monotonitás miatt \(\displaystyle f\left(f(x)-{1\over x}+1\right)\ge1\), ugyanakkor xf(x)<1, ami szintén ellentmondás. Az f(x) értéke tehát nem lehet más, csak \(\displaystyle {1\over x}\). Az állítást ezzel igazoltuk x>1-re.

Legyen most x>0 tetszőleges, és y=1. Az eredményt alkalmazhatjuk az f(x)+1 és 1+xy számokra, mert ezek nagyobbak 1-nél. Így a függvényegyenlet szerint

\(\displaystyle f(f(x)+1)={1\over f(x)+1}=xf(1+x)={x\over1+x},\)

\(\displaystyle f(x)={1+x\over x}-1={1\over x}.\)

4. állítás. Az f(x)=1/x függvényre teljesül a függvényegyenlet.

Biz. f(f(x)+y)=f(1/x+y)=f((1+xy)/x)=x/(1+xy)=xf(1+xy).

Írd meg véleményedet a feladatokról!